[Rozgrzewka OM][MIX][Nierówności] Nierówności

Zadania z kółek matematycznych lub obozów przygotowujących do OM. Problemy z minionych olimpiad i konkursów matematycznych.
Regulamin forum
Wszystkie tematy znajdujące się w tym dziale powinny być tagowane tj. posiadać przedrostek postaci [Nierówności], [Planimetria], itp.. Temat może posiadać wiele różnych tagów. Nazwa tematu nie może składać się z samych tagów.
bosa_Nike
Użytkownik
Użytkownik
Posty: 1664
Rejestracja: 16 cze 2006, o 15:40
Płeć: Kobieta
Podziękował: 71 razy
Pomógł: 445 razy

Re: [Rozgrzewka OM][MIX][Nierówności] Nierówności

Post autor: bosa_Nike »

@albanczyk123456 - spróbuj zrobić takie zadanie, tyle że przy `n=3` i słabej nierówności.
Awatar użytkownika
Sylwek
Użytkownik
Użytkownik
Posty: 2716
Rejestracja: 21 maja 2007, o 14:24
Płeć: Mężczyzna
Lokalizacja: Warszawa
Podziękował: 160 razy
Pomógł: 657 razy

Re: [Rozgrzewka OM][MIX][Nierówności] Nierówności

Post autor: Sylwek »

Premislav pisze: 23 sie 2019, o 00:00Nowe zadanie:
w rzeczywistych dodatnich \(\displaystyle{ x_1, x_2, \ldots x_n}\) (\(\displaystyle{ n>3}\)) spełniających warunek \(\displaystyle{ x_1 x_2\ldots x_n=1}\) proszę udowodnić, że
\(\displaystyle{ \frac{1}{1+x_1+x_1x_2} +\frac{1}{1+x_2+x_2x_3}+\ldots+\frac{1}{1+x_n+x_n x_1}>1}\)
Z racji warunku na iloczyn, można podstawić \(\displaystyle{ x_i=\frac{a_{i+1}}{a_i}}\) dla pewnych liczb \(\displaystyle{ a_i>0}\). Przekształćmy lewą stronę:
\(\displaystyle{ L=\frac{1}{1+x_1+x_1x_2} +\frac{1}{1+x_2+x_2x_3}+\ldots+\frac{1}{1+x_n+x_n x_1}= \\
=\frac{1}{1+\frac{a_2}{a_1}+\frac{a_2}{a_1}\frac{a_3}{a_2}} +\frac{1}{1+\frac{a_3}{a_2}+\frac{a_3}{a_2}\frac{a_4}{a_3}}+\ldots+\frac{1}{1+\frac{a_1}{a_n}+\frac{a_1}{a_n}\frac{a_2}{a_1}}= \\
=\frac{a_1}{a_1+a_2+a_3} +\frac{a_2}{a_2+a_3+a_4}+\ldots+\frac{a_n}{a_n+a_1+a_2}.}\)


Ale z racji \(\displaystyle{ n>3}\) każdy mianownik jest ściśle mniejszy od \(\displaystyle{ S=a_1+a_2+\ldots+a_n}\), zatem
\(\displaystyle{ L>\frac{a_1}{S} +\frac{a_2}{S}+\ldots+\frac{a_n}{S}=\frac{S}{S}=1,}\)
co kończy dowód.
bosa_Nike pisze: 22 wrz 2019, o 00:37 @albanczyk123456 - spróbuj zrobić takie zadanie, tyle że przy \(\displaystyle{ n=3}\) i słabej nierówności.
Co miałaś na myśli? Jakaś indukcja lub wykorzystanie mniejszych \(\displaystyle{ n}\) przy dowodzie dla większych indeksów?

Nowe zadanie:
Udowodnij, że dla \(\displaystyle{ x_i>0}\) zachodzi nierówność
\(\displaystyle{ \frac{x_1^3}{x_1^2+x_1x_2+x_2^2}+\frac{x_2^3}{x_2^2+x_2x_3+x_3^2}+\ldots+\frac{x_n^3}{x_n^2+x_nx_1+x_1^2} \ge \frac{x_1+x_2+\ldots+x_n}{3}}\)
Awatar użytkownika
Premislav
Użytkownik
Użytkownik
Posty: 15687
Rejestracja: 17 sie 2012, o 13:12
Płeć: Mężczyzna
Lokalizacja: Warszawa
Podziękował: 195 razy
Pomógł: 5220 razy

Re: [Rozgrzewka OM][MIX][Nierówności] Nierówności

Post autor: Premislav »

Lemat:
dla \(\displaystyle{ a,b>0}\) zachodzi nierówność
\(\displaystyle{ \frac{a^{3}}{a^{2}+ab+b^{2}}\ge \frac{2a-b}{3}}\).
Dowód lematu:
równoważnie mamy
\(\displaystyle{ 3a^{3}\ge (2a-b)\left(a^{2}+ab+b^{2}\right)\\ 3a^{3}\ge 2a^{3}+2a^{2}b+2ab^{2}-ba^{2}-ab^{2}-b^{3}\\a^{3}+b^{3}-a^{2}b-ab^{2}\ge 0\\(a+b)(a-b)^{2}\ge 0}\)
co jest już oczywiste wobec nieujemności kwadratu liczby rzeczywistej.

Dodajemy tezę lematu kolejno dla \(\displaystyle{ a=x_{i}. \ b=x_{i+1}, \ i=1,2\ldots n}\), gdzie przyjmujemy \(\displaystyle{ x_{n+1}=x_{1}}\), co kończy dowód.

Nowe:
dla rzeczywistych \(\displaystyle{ a_{1}, a_{2}\ldots a_{99}}\) spełniających warunek
\(\displaystyle{ (1+a_{1})(1+a_{2})\ldots(1+a_{99})=100}\)
proszę wykazać, że
\(\displaystyle{ \left(1+a_{1}^{2}\right)\left(1+2a_{2}^{2}\right)\ldots \left(1+99a_{99}^{2}\right)\ge 100}\).
bosa_Nike
Użytkownik
Użytkownik
Posty: 1664
Rejestracja: 16 cze 2006, o 15:40
Płeć: Kobieta
Podziękował: 71 razy
Pomógł: 445 razy

Re: [Rozgrzewka OM][MIX][Nierówności] Nierówności

Post autor: bosa_Nike »

Zahionie, przybądź. :wink:

@Sylwek
Ukryta treść:    
Zahion
Moderator
Moderator
Posty: 2095
Rejestracja: 9 gru 2012, o 19:46
Płeć: Mężczyzna
Lokalizacja: Warszawa, mazowieckie
Podziękował: 139 razy
Pomógł: 504 razy

Re: [Rozgrzewka OM][MIX][Nierówności] Nierówności

Post autor: Zahion »

bosa_Nike pisze: 15 paź 2019, o 17:47 Zahionie, przybądź. :wink:
:wink:

Co do ostatniego zadania, to moja idea jest taka:

\(\displaystyle{ (1+a^{2})(1+2b^{2})(1+3c^{2}) = [\frac{1}{2}\left( 1+a\right)^{2} + \frac{1}{2}(\left( 1 - a\right)^{2}][...][( \frac{3}{4}\left( 1+c\right)^{2} + \frac{1}{4}\left( 1-3c\right)^{2}] }\)

Teraz to z rozszerzonego CS da nam:
\(\displaystyle{ L \ge ( \sqrt[3]{[ \frac{1}{4} (1+a)(1+b)(1+c)]^{2}} + |x| )^{3} \ge \frac{1}{4} [(1+a)(1+b)(1+c)]^{2} = 4 }\)

przy czym \(\displaystyle{ |x| }\) to wartość bezwględna iloczynów pozostałych wyrazów do potęgi 1/3.

Jeżeli poprawnie, to postaram się coś wrzucić dziś.
Awatar użytkownika
timon92
Użytkownik
Użytkownik
Posty: 1657
Rejestracja: 6 paź 2008, o 16:47
Płeć: Mężczyzna
Lokalizacja: Katowice
Podziękował: 7 razy
Pomógł: 472 razy

Re: [Rozgrzewka OM][MIX][Nierówności] Nierówności

Post autor: timon92 »

@up prościej byłoby po prostu szacować ze Szwarca \(\displaystyle{ (1+ka_k^2)(1+\frac 1k) \ge (1+a_k)^2}\) i wymnożyć po wszystkich \(\displaystyle{ k}\)
Awatar użytkownika
Premislav
Użytkownik
Użytkownik
Posty: 15687
Rejestracja: 17 sie 2012, o 13:12
Płeć: Mężczyzna
Lokalizacja: Warszawa
Podziękował: 195 razy
Pomógł: 5220 razy

Re: [Rozgrzewka OM][MIX][Nierówności] Nierówności

Post autor: Premislav »

Idea jest dobra, choć spodziewałem się rozwiązania, które podał timon (to jest zadanie z obozu Olimpiady Matematycznej (jeszcze wtedy) Gimnazjalistów, poziom OM z któregoś roku, bodajże 2014, i rozwiązanie wzorcowe właśnie tak wyglądało). Możesz wrzucać nowe zadanie.
Zahion
Moderator
Moderator
Posty: 2095
Rejestracja: 9 gru 2012, o 19:46
Płeć: Mężczyzna
Lokalizacja: Warszawa, mazowieckie
Podziękował: 139 razy
Pomógł: 504 razy

Re: [Rozgrzewka OM][MIX][Nierówności] Nierówności

Post autor: Zahion »

Jeżeli coś mi dzisiaj wpadnie do głowy to wrzucę, w przeciwnym wypadku, żeby nie blokować będzie wolne.
Awatar użytkownika
Premislav
Użytkownik
Użytkownik
Posty: 15687
Rejestracja: 17 sie 2012, o 13:12
Płeć: Mężczyzna
Lokalizacja: Warszawa
Podziękował: 195 razy
Pomógł: 5220 razy

Re: [Rozgrzewka OM][MIX][Nierówności] Nierówności

Post autor: Premislav »

Może czas coś wrzucić.
Przy ustalonych liczbach całkowitych dodatnich \(\displaystyle{ n,k}\) spełniających warunek \(\displaystyle{ n\ge 4k}\) proszę znaleźć najmniejsze takie \(\displaystyle{ \lambda(n,k)}\), że dla dowolnych liczb rzeczywistych dodatnich \(\displaystyle{ a_{1},a_{2}, \ldots a_{n}}\) zachodzi nierówność
\(\displaystyle{ \sum_{i=1}^{n}\frac{a_{i}}{\sqrt{a_{i}^{2}+a_{i+1}^{2}+\ldots+a_{i+k}^{2}}}\le \lambda}\)
przy czym przyjmujemy, że \(\displaystyle{ a_{n+i}=a_{i}}\) dla \(\displaystyle{ i=1,2\ldots k}\).
Awatar użytkownika
Premislav
Użytkownik
Użytkownik
Posty: 15687
Rejestracja: 17 sie 2012, o 13:12
Płeć: Mężczyzna
Lokalizacja: Warszawa
Podziękował: 195 razy
Pomógł: 5220 razy

Re: [Rozgrzewka OM][MIX][Nierówności] Nierówności

Post autor: Premislav »

hint:    
Awatar użytkownika
Premislav
Użytkownik
Użytkownik
Posty: 15687
Rejestracja: 17 sie 2012, o 13:12
Płeć: Mężczyzna
Lokalizacja: Warszawa
Podziękował: 195 razy
Pomógł: 5220 razy

Re: [Rozgrzewka OM][MIX][Nierówności] Nierówności

Post autor: Premislav »

Dobra, wyjebongo, nie chce mi się pisać rozwiązania.


Nowe zadanie: dla \(\displaystyle{ a,b,c}\) różnych od zera proszę udowodnić, że zachodzi nierówność
\(\displaystyle{ 2b^{2}+\frac{1}{a^{2}}+\frac{1}{c^{2}}+\frac{b}{a}+\frac{b}{c}+\sqrt{2a^{4}+2c^{4}}\ge 2\sqrt{3} }\)
bosa_Nike
Użytkownik
Użytkownik
Posty: 1664
Rejestracja: 16 cze 2006, o 15:40
Płeć: Kobieta
Podziękował: 71 razy
Pomógł: 445 razy

Re: [Rozgrzewka OM][MIX][Nierówności] Nierówności

Post autor: bosa_Nike »

Wetnę się, bo mam taką akuratną, w sam raz na następne zadanie.
Ukryta treść:    
Dla `a,b,c>0` oraz stałej `k\ge 2` pokaż, że $$\left(a^2+k\right)\left(b^2+k\right)\left(c^2+k\right)\ge (k+1)(a+b+c+k-2)^2+(abc-1)^2$$
Awatar użytkownika
Premislav
Użytkownik
Użytkownik
Posty: 15687
Rejestracja: 17 sie 2012, o 13:12
Płeć: Mężczyzna
Lokalizacja: Warszawa
Podziękował: 195 razy
Pomógł: 5220 razy

Re: [Rozgrzewka OM][MIX][Nierówności] Nierówności

Post autor: Premislav »

Ukryta treść:    
Jeśli nie ma błędu, to z uwagi na porę i brak lepszego pomysłu z mojej strony następny może być ten lemat z poprzedniego niezrobionego zadania, tj. proszę udowodnić, że w dodatnich mamy
\(\displaystyle{ \frac{a}{\sqrt{a^{2}+b^{2}}}+\frac{b}{\sqrt{b^{2}+c^{2}}}+\frac{c}{\sqrt{c^{2}+d^{2}}}+\frac{d}{\sqrt{d^{2}+a^{2}}}\le 3}\)
(NB równość nie zachodzi).
bosa_Nike
Użytkownik
Użytkownik
Posty: 1664
Rejestracja: 16 cze 2006, o 15:40
Płeć: Kobieta
Podziękował: 71 razy
Pomógł: 445 razy

Re: [Rozgrzewka OM][MIX][Nierówności] Nierówności

Post autor: bosa_Nike »

Ukryta treść:    
Jeżeli powyższe wystarczy, to prawo do zamieszczenia nowego zadania przekazuję dowolnej chętnej osobie.


PS Jestem pod wielkim wrażeniem oficjalnej (a jakże!) strony konkursu Baltic Way 2019. (Baltic) Way to go! :cry:
Awatar użytkownika
timon92
Użytkownik
Użytkownik
Posty: 1657
Rejestracja: 6 paź 2008, o 16:47
Płeć: Mężczyzna
Lokalizacja: Katowice
Podziękował: 7 razy
Pomógł: 472 razy

Re: [Rozgrzewka OM][MIX][Nierówności] Nierówności

Post autor: timon92 »

może tak:    
Dodano po 18 godzinach 17 minutach 25 sekundach:
a może jednak nie tak XD palnąłem głupotę, jakikolwiek Karamata daje szacowanie w złą stronę XD
ODPOWIEDZ